Finding an upper bound that is not the supremum

Click For Summary
SUMMARY

The discussion centers on the concept of upper bounds in mathematical sets, specifically the interval (0,1). It is established that while the supremum of the set S is 1, any number greater than 1, such as 2, qualifies as an upper bound. The definition of supremum as the least upper bound is emphasized, confirming that all other upper bounds must exceed this value.

PREREQUISITES
  • Understanding of mathematical intervals and sets
  • Familiarity with the concept of supremum in real analysis
  • Basic knowledge of upper and lower bounds
  • Ability to interpret mathematical definitions and constructions
NEXT STEPS
  • Study the properties of supremum and infimum in real analysis
  • Explore examples of upper and lower bounds in various mathematical contexts
  • Learn about the completeness property of the real numbers
  • Investigate the implications of upper bounds in optimization problems
USEFUL FOR

Mathematicians, students of real analysis, and anyone interested in understanding the concepts of upper bounds and supremum in mathematical sets.

ver_mathstats
Messages
258
Reaction score
21
Homework Statement
Let S=(0,1), find a real number M which is an upper bound of S, but M is not the supremum of S.
Relevant Equations
S=(0,1)
I just want to see if I did this correctly, the interval (0,1) has 2 as an upper bound but the supremum of S is 1. So M would be equal to 2?

Thank you.
 
Physics news on Phys.org
Yes, any number greater than 1 would do.
 
PeroK said:
Yes, any number greater than 1 would do.
Okay thank you.
 
Just to expand a bit. The Supremum is , by definition/construction, the _Least_ Upper Bound. This implies any other upper bound should be greater than the Sup, as @PeroK wrote.
 
Question: A clock's minute hand has length 4 and its hour hand has length 3. What is the distance between the tips at the moment when it is increasing most rapidly?(Putnam Exam Question) Answer: Making assumption that both the hands moves at constant angular velocities, the answer is ## \sqrt{7} .## But don't you think this assumption is somewhat doubtful and wrong?

Similar threads

  • · Replies 2 ·
Replies
2
Views
816
  • · Replies 2 ·
Replies
2
Views
2K
  • · Replies 7 ·
Replies
7
Views
3K
  • · Replies 1 ·
Replies
1
Views
2K
Replies
6
Views
3K
  • · Replies 1 ·
Replies
1
Views
2K
  • · Replies 3 ·
Replies
3
Views
2K
  • · Replies 13 ·
Replies
13
Views
2K
  • · Replies 1 ·
Replies
1
Views
1K
  • · Replies 7 ·
Replies
7
Views
3K